LSAT and Law School Admissions Forum

Get expert LSAT preparation and law school admissions advice from PowerScore Test Preparation.

User avatar
 Dave Killoran
PowerScore Staff
  • PowerScore Staff
  • Posts: 5853
  • Joined: Mar 25, 2011
|
#84896
Complete Question Explanation
(The complete setup for this game can be found here: lsat/viewtopic.php?t=8333)

The correct answer choice is (D)

This question asks you to suspend the second rule and replace it with an answer choice that will have an identical effect on the order of the artifacts.

Answer choice (D) fulfills this role by placing N, J, and F ahead of T, which mirrors the situation shown in the combination of the first two rules. In those two rules, N and J are older than T (which is the second rule we are attempting to replace), and from the first rule F is older than J. Thus, when this rule states that N, J, and F are older than T, that perfectly matches the effect of the second rule.
 netherlands
  • Posts: 136
  • Joined: Apr 17, 2013
|
#8806
Hi there Powerscore,

I just had a question about #11 on this one. I answered this question correctly, but am still not sure whether I was interpreting it/approaching it correctly.

I basically tried to find whatever setup would leave "T" in slot 4. I guess it just felt like the only concrete characteristic that I could find and then try to match.

Also,I'm not even sure how to interpret the effect that answer choices "C" and "E" would leave on the game. The only thing they seem to do ( especially with the "if and only if" clauses) is make the setup more vague - bc of course, without more premises we don't know if either of the "if and only if" clauses is selected. So maybe thats an answer to part of my question in itself - If I'm trying to select a premise that matches the ordering effect of the diagram - then those that make it more vague (whether or not I really understand or even dig into the outcomes) are definitely not answer choices. If that's a proper way to go about it then I can see how I'd eliminate C and E.

Answer choice B seemed to place "T" in slot 3 - which was how I eliminated it.
And answer choice A places it as early as slot 2.

Answer choice "D" immediately places "T" back in four.

Is this how you would have recommended we approach the game?

Thanks ahead of time! :)
 Nikki Siclunov
PowerScore Staff
  • PowerScore Staff
  • Posts: 1362
  • Joined: Aug 02, 2011
|
#8852
Hi netherlands,

Simply ensuring that T is 4 will not replicate the effect of the suspended rule (N and J are older than T). This is because without the rule in question, other variables could potentially fill in the first three slots. Our job, more precisely, is to identify a rule ensuring that three particular variables -- F, J, and N -- are older than T. Answer choice (D) accomplishes this goal, and is therefore correct.

Here's why (C) is incorrect (the fact that the rule uses conditional language is not, by itself, enough to eliminate it from consideration):

According to the rule in answer choice (C), N > T :dbl: J > T. This rule suggests two possible sequences for N, T, and J: on one hand, you can have N and J > T (which is what we are looking to replicate). However, due to the conditional nature of this rule, we have the following contrapositive to take into account: T > N :dbl: T > J. So, the other possible sequence is T > N and J. This broadens the scope of the rule we are trying to replace, and is therefore incorrect.

Hope this helps... let me know if it does!

Thanks!
 netherlands
  • Posts: 136
  • Joined: Apr 17, 2013
|
#8896
Hi there,

So basically, the question is asking which substituted rule would maintain the already allowed order of all of the artifacts?

Basically, a rule that doesn't distort the placement that's already been made possible by the first set of rules. And this applies to each and every variable.
 Nikki Siclunov
PowerScore Staff
  • PowerScore Staff
  • Posts: 1362
  • Joined: Aug 02, 2011
|
#8921
You got it! I couldn't have said it better myself. :-)
 ronibass
  • Posts: 15
  • Joined: Jun 18, 2019
|
#71015
I chose B for this question. For my set up I had the order as F-J-T and N-T. So then wouldn't answer choice B suffice? It ensures that F and N are in front of T which is the same as how I originally had it. If not then what am I missing?
User avatar
 KelseyWoods
PowerScore Staff
  • PowerScore Staff
  • Posts: 1079
  • Joined: Jun 26, 2013
|
#71033
Hi Ronibass!

Answer choice (B) preserves the relationship that both F and N are before T, but it loses the J :longline: T relationship which we also need to make sure is present.

Hope this helps!

Best,
Kelsey
 ser219
  • Posts: 20
  • Joined: Sep 05, 2019
|
#71406
I got this question correct but D confused me. It says that all are older than T, except H&P. Couldn't P be older than T in the original realm of possibilities? PFHJNT or FJNHPT could work right?
User avatar
 KelseyWoods
PowerScore Staff
  • PowerScore Staff
  • Posts: 1079
  • Joined: Jun 26, 2013
|
#71420
Hi ser219!

Answer choice (D) says that all of the artifacts except the headdress and the plaque MUST be older than the tureen. As you said, the plaque could be older than the tureen, but it doesn't have to be. Answer choice (D) isn't saying that the plaque cannot be older than the tureen, just that it isn't required to be older than the tureen.

Hope this helps!

Best,
Kelsey
User avatar
 bebeg3168
  • Posts: 22
  • Joined: Aug 01, 2022
|
#96495
Hi I having a hard time wrapping my head around this one. I look at the diagrams I drew out and they match what is on this forum for this question. I see how the plaque could be older however I'm not seeing the headdress as older. My diagram looks like this:


:arrow: J
F
:arrow: H :arrow: P

N :larrow:

So the answer is H & P must be older than the Tureen What about J and N? :-? I'm sorry for confusion.

Get the most out of your LSAT Prep Plus subscription.

Analyze and track your performance with our Testing and Analytics Package.